LSAT and Law School Admissions Forum

Get expert LSAT preparation and law school admissions advice from PowerScore Test Preparation.

User avatar
 Dave Killoran
PowerScore Staff
  • PowerScore Staff
  • Posts: 5852
  • Joined: Mar 25, 2011
|
#44074
Complete Question Explanation
(The complete setup for this game can be found here: lsat/viewtopic.php?t=6217)

The correct answer choice is (C)

This question is based purely on the distribution possibilities.

Answer choice (A) is incorrect because V could carry only 1 type, and T carries 2 types.

Answer choice (B) is incorrect because V or Z could carry 2 types while the other carries only 1.

Answer choice (C) is the correct answer choice.

Answer choice (D) is incorrect because T carries 2 types and Z cannot carry more than 2 types.

Answer choice (E) is incorrect because X carries exactly one more type than S carries.

Get the most out of your LSAT Prep Plus subscription.

Analyze and track your performance with our Testing and Analytics Package.